Finding multivariate extrema with degenerate hessian matrix

Click For Summary
The discussion revolves around determining the conditions under which the function f(x,y) = ax^3 + by^3 + cx^4 + dy^4 - (x+y)^5 has a local minimum at (0,0). The gradient at (0,0) is zero for any parameters, but the Hessian matrix is also zero, complicating the analysis of critical points. Participants suggest examining the function along specific lines through the origin to establish local minima, though there is debate about the sufficiency of these conditions. Concerns are raised regarding the differentiability of the function at (0,0) and the validity of using certain mathematical methods. Overall, the conversation highlights the challenges of analyzing critical points when standard tools are ineffective.
talolard
Messages
119
Reaction score
0

Homework Statement


For what real values of the parameters a,b,c,d does the functiob f(x,y)=ax^3+by^3+cx^4+dy^4-(x+y)^5 have a local minimum at (0,0)

Homework Equations


I calculated the gradient at (0,0) and it is always zero regardless of parameters.
The problem is that the Hessian matrix is also zero so I don't know what kind of criticial point it is.
I also noticed that if (0,0) is a minimum then ax^3+by^3+cx^4+dy^4>(x+y)^5 in the nieberhood but that still hasnt taken me very far.
I don't see how I can use Lagrange multipliers, the inverse or implicit function theorems, since the gradient is 0 which precludes using them in any direct way. So my arsenal seems rather depleted.
Any ideas?
Thanks
Tal

The Attempt at a Solution

 
Physics news on Phys.org
Hi talolard! :smile:

Your Hessian matrix is indeed zero, which means that we can't use it here. We'll have to use more direct methods.

Consider the lines y=\alpha x and x=0 through (0,0). If f has a local minimum in (0,0), then it must also be a local minimum on these lines. So it suffices to show that f(x,\alpha x) and f(0,y) have local minima in 0. And this is easy since the function has only one variable!
 
micromass said:
Hi talolard! :smile:

Your Hessian matrix is indeed zero, which means that we can't use it here. We'll have to use more direct methods.

Consider the lines y=\alpha x and x=0 through (0,0). If f has a local minimum in (0,0), then it must also be a local minimum on these lines. So it suffices to show that f(x,\alpha x) and f(0,y) have local minima in 0. And this is easy since the function has only one variable!

Perhaps I misunderstand your point, but those conditions are necessary but not sufficient to guarantee a local minimum.

f(x,y) = (y - x2)(y-2x2) has a local min along all rays through the origin but has a saddle point there.
 
LCKurtz said:
Perhaps I misunderstand your point, but those conditions are necessary but not sufficient to guarantee a local minimum.

f(x,y) = (y - x2)(y-2x2) has a local min along all rays through the origin but has a saddle point there.

I see your point, and I must admit I haven't really thought about it. But the point is that your f is not differentiable in 0 (I think). I guess that the method would work for differentiable maps (although I can't seem to find a proof for it now). Correct me if I'm wrong!

Edit: thinking some more about it leads me to think that it is not even true for differentiable maps. Can't really find a counterexample, but I don't think this is the way to prove it.
 
Last edited:
micromass said:
I see your point, and I must admit I haven't really thought about it. But the point is that your f is not differentiable in 0 (I think). I guess that the method would work for differentiable maps (although I can't seem to find a proof for it now). Correct me if I'm wrong!

Edit: thinking some more about it leads me to think that it is not even true for differentiable maps. Can't really find a counterexample, but I don't think this is the way to prove it.
No: the above f is just a product of polynomials, so is as differentiable as you could ever want.

RGV
 
Question: A clock's minute hand has length 4 and its hour hand has length 3. What is the distance between the tips at the moment when it is increasing most rapidly?(Putnam Exam Question) Answer: Making assumption that both the hands moves at constant angular velocities, the answer is ## \sqrt{7} .## But don't you think this assumption is somewhat doubtful and wrong?

Similar threads

Replies
4
Views
2K
  • · Replies 8 ·
Replies
8
Views
4K
  • · Replies 7 ·
Replies
7
Views
3K
  • · Replies 5 ·
Replies
5
Views
2K
  • · Replies 4 ·
Replies
4
Views
4K
  • · Replies 2 ·
Replies
2
Views
2K
Replies
10
Views
1K
  • · Replies 11 ·
Replies
11
Views
2K
  • · Replies 4 ·
Replies
4
Views
2K
  • · Replies 8 ·
Replies
8
Views
2K